How does the probability density function handle infinity in integrals?

Click For Summary
The discussion centers on handling integrals involving infinity, specifically the integral of the function x^(-2.5). It is clarified that when evaluating the integral from 1 to infinity, one must take the limit as the upper bound approaches infinity. The correct formulation is noted as ∫_1^∞ x^(-2.5) dx, which converges to 2/3. The importance of correctly identifying the limits in such integrals is emphasized to avoid divergence. Understanding these concepts is crucial for accurate calculations in probability density functions.
converting1
Messages
65
Reaction score
0
http://gyazo.com/02812d5d8f1d07c72153c9f66740e147

I've dealt with integrals with infinity before. When considering the part x >= 1 , do I take the limit as if it's a very large number? i.e. ## \int_0^{\infty} x^{-2.5} \ dx = 2/3 ## ?
 
Physics news on Phys.org
Yes, take the limit as the upper value goes to infinity.
 
converting1 said:
http://gyazo.com/02812d5d8f1d07c72153c9f66740e147

I've dealt with integrals with infinity before. When considering the part x >= 1 , do I take the limit as if it's a very large number? i.e. ## \int_0^{\infty} x^{-2.5} \ dx = 2/3 ## ?

You mean ##\int_1^\infty##. What you have written would diverge.
 
ok thank you
 
Question: A clock's minute hand has length 4 and its hour hand has length 3. What is the distance between the tips at the moment when it is increasing most rapidly?(Putnam Exam Question) Answer: Making assumption that both the hands moves at constant angular velocities, the answer is ## \sqrt{7} .## But don't you think this assumption is somewhat doubtful and wrong?

Similar threads

  • · Replies 19 ·
Replies
19
Views
4K
  • · Replies 1 ·
Replies
1
Views
2K
  • · Replies 4 ·
Replies
4
Views
4K
  • · Replies 4 ·
Replies
4
Views
2K
  • · Replies 6 ·
Replies
6
Views
2K
Replies
17
Views
2K
  • · Replies 2 ·
Replies
2
Views
2K
  • · Replies 4 ·
Replies
4
Views
2K
  • · Replies 3 ·
Replies
3
Views
2K
Replies
5
Views
2K